esercizi su massimi e minimi in 2 variabili

Questa pagina è dedicata a fornirti delle metodologie per lo svolgimento di Esercizi su massimi e minimi in 2 variabili. Quindi mettiti comodo e segui i passaggi con attenzione perché possono esserti molto utili per il tuo esame di analisi matematica. Questi esercizi sono svolti con cura e attenzione ai dettagli, ma anche con un pizzico di originalità mostrando quelli che sono i principali metodi per approcciare a questo tipo di problema.

Esercizi su massimi e minimi in 2 variabili su insiemi compatti (METODO DELLE CURVE DI LIVELLO)

Per cominciare propongo un metodo un pò inusuale, ma molto intuitivo ed elegante, che è quello del metodo delle curve di livello per risolvere Esercizi su massimi e minimi in 2 variabili. Il metodo consiste nel capire in che direzione crescono le curve di livello del grafico e in quali punti intercettano l’insieme compatto su cui stiamo cercando massimi e minimi.

ESERCIZIO 1

Cercare massimo e minimo assoluto di$f\left( x,y \right)={{x}^{2}}+{{y}^{2}}-4x$ sull’insieme $A=\left\{ \left( x,y \right)\in {{\mathbb{R}}^{2}}:\,\,1\le y\le {{e}^{x}},\,\,\,x\le 3 \right\}$ .

Le curve di livello della funzione $f\left( x,y \right)$sono circonferenze ${{x}^{2}}+{{y}^{2}}-4x=k$

${{x}^{2}}-4x+4+{{y}^{2}}=k+4$, ${{\left( x-2 \right)}^{2}}+{{y}^{2}}={{\left( \sqrt{k+4} \right)}^{2}}$, sono circonferenze di centro $C=\left( 2,0 \right)$ e raggio $R=\sqrt{4+k}$ .

All’aumentare di $k$ aumenta anche il raggio della circonferenza. Pertanto bisogna individuare il punto a minima distanza da $C$ ovvero ${{P}_{0}}$ e il punto a massima distanza cioè ${{P}_{1}}$.

In ${{P}_{0}}$ la circonferenza è tangente alla retta di equazione $y=1$ .

${{x}^{2}}+{{1}^{2}}-4x=k\,\,\,\Rightarrow \,\,\,{{x}^{2}}-4x+1-k\,\,=0$ . La circonferenza è tangente alla retta se $\Delta =0\,\,\,\Rightarrow \,\,\,16-4+4k=0\,\,\,\Rightarrow \,\,\,k=-3$ . Pertanto $\underset{x\in A}{\mathop{\min }}\,f=-3$

Il punto a massima distanza è in ${{P}_{1}}=\left( 3,{{e}^{3}} \right)$ , punto di intersezione tra la retta e l’esponenziale.

$\underset{x\in A}{\mathop{\max }}\,f=f\left( {{P}_{1}} \right)=9+{{e}^{6}}-12={{e}^{6}}-3$

Esercizi su massimi e minimi in 2 variabili - metodo delle curve di livello

Nella figura quelle che vediamo colorate in rosso sono le curve di livello, mentre in verde c’è l’insieme del piano su cui andiamo a cercare massimi e minimi.

massimi e minimi su compatti – metodo delle curve di livello

ESERCIZIO 2

$f\left( x,y \right)=\frac{x-y+1}{x+3}$ su $A=\left\{ \left( x,y \right)\in {{\mathbb{R}}^{2}}:\,\,2-\frac{2}{3}{{x}^{2}}\le y\le 3-{{x}^{2}} \right\}$

L’insieme A è rappresentato geometricamente dallo spazio compreso tra due parabole. I punti di intersezioni tra esse sono $2-\frac{2}{3}{{x}^{2}}=3-{{x}^{2}}$ $\Rightarrow$ ${{x}^{2}}=3$ $\Rightarrow $ $x=\pm \sqrt{3}$ , $y=0$

Le curve di livello di $f$ sono un fascio di rette proprio $\frac{x-y+1}{x+3}=k$

Da cui: $x-y+1=k\left( x+3 \right)$. Il centro del fascio è dato dalla soluzione del sistema:

$\left\{ \begin{aligned} & x – y + 1 = 0 \\ & x + 3 = 0 \end{aligned} \right. \Rightarrow C = (-3, -2)$

Inoltre esplicitando l’equazione rispetto a $y$ si ha che $y=\left( 1-k \right)x+1-3k$ . Quindi il coefficiente angolare della retta è legata alla quota $k$ tramite $m=1-k$ . Quindi al crescere del coefficiente angolare diminuisce la quota della funzione.

Esercizi su massimi e minimi in 2 variabili - METODO DELLE CURVE DI LIVELLO.

metodo delle curve di livello per individuare massimi e vinimi

La quota minima si ha per in corrispondenza della retta del fascio passante per il punto $\left( \sqrt{3},0 \right)$ . Quindi $\underset{\left( x,y \right)\in A}{\mathop{\max }}\,f={{k}_{\min }}=\frac{\sqrt{3}+1}{\sqrt{3}+3}=\frac{\sqrt{3}+1}{\sqrt{3}\left( 1+\sqrt{3} \right)}=\frac{1}{\sqrt{3}}$

La quota massima si in corrispondenza della retta tangente alla parabola più alta. Bisogna porre $y=\left( 1-k \right)x+1-3k$nell’equazione della parabola $y=3-{{x}^{2}}$e imporre $\Delta =0$ .

$\left( 1-k \right)x+1-3k=3-{{x}^{2}}\,\,\Rightarrow \,\,{{x}^{2}}+\left( 1-k \right)x-2-3k=0\,\,\,\Rightarrow \,\,\,$

$\Delta =1+{{k}^{2}}-2k+8+12k={{k}^{2}}+10k+9=0\,\,\Rightarrow \,\underset{\left( x,y \right)\in A}{\mathop{\min }}\,f=\,\,{{k}_{\min }}=-1$

ESERCIZIO 3

$f\left( x,y \right)=\log \left( x+1 \right)+\log \left( y+1 \right)$ su $A=\{ \left( x,y \right)\in {{\mathbb{R}}^{2}}:\,\,{{x}^{2}}+{{y}^{2}}\le 4,x\ge 0,y\ge 0 \}$

L’insieme A è un arco circonferenza delimitato dai semiassi positivi delle ascisse e delle ordinate.

$f\left( x,y \right)=\log \left( x+1 \right)+\log \left( y+1 \right)=\log \left[ \left( x+1 \right)\left( y+1 \right) \right]$.

Il logaritmo è una funzione monotona, quindi le curve di livello della funzione $f\left( x,y \right)$ crescono nella stessa direzione in cui crescono quelle del suo argomento, cioè se ${{c}_{2}}>{{c}_{1}}\,\,\,\Rightarrow \,\,\,\log {{c}_{2}}>\log {{c}_{1}}$ .

Cerchiamo di capire cosa rappresentano le curve di livello dell’argomento del logaritmo:

$\left( x+1 \right)\left( y+1 \right)=c$ può essere vista come una traslazione del luogo geometrico definito in maniera implicita dall’equazione $xy=c$, di un vettore $v=\left( -1,-1 \right)$ .

$xy=c$ è una iperbole avente come asintoti gli assi cartesiani e posizionata nel primo e terzo quadrante se $c>0$, nel secondo e quarto quadrante $c<0$. Se invece $c=0$ l’iperbole degenera in due rette (asse x e asse y).

In particolare siamo interessati al caso $c>0$

Esercizi su massimi e minimi in 2 variabili - METODO DELLE CURVE DI LIVELLO

metodo delle curve di livello per individuare massimi e minimi

Al crescere di $c$ le curve di livello si spostano verso l’alto, e il vertice delle iperboli si sposta lungo la bisettrice del primo e terzo quadrante. Il massimo della funzione sull’insieme $A$ (tenendo presente l’andamento delle curve di livello in figura e le simmetrie del problema), si ha sulla circonferenza in coordinate ${{P}_{\max }}=\left( \sqrt{2},\,\,\sqrt{2} \right)$ e vale $f\left( {{P}_{\max }} \right)=\log \left( 5+\sqrt{2} \right)$ , mentre il minimo si trova nell’origine e vale $f\left( 0,0 \right)=0$

Esercizi su massimi e minimi in 2 variabili (Metodo della Parametrizzazione del bordo)

ESERCIZIO 4

Data la funzione $f\left( x,y \right)=y={{x}^{2}}+{{y}^{2}}-\frac{x}{2}$ cercare massimo e minimo assoluti della funzione su $C=\{{{x}^{2}}+{{y}^{2}}\le 1,\,\,\,{{\left( x-1 \right)}^{2}}+{{\left( y+1 \right)}^{2}}\le 1 \}$

L’insieme $C$ è costituito dall’interno di una circonferenza centrata nell’origine e di raggio unitario e l’esterno di una circonferenza di raggio unitario e centrata in $c=\left( 1,-1 \right)$ .

Cerchiamo i punti stazionari: $\nabla f=0$ :

$\left\{ \begin{aligned} & 2x – \frac{1}{2} = 0 \\ & 2y = 0 \end{aligned} \right. \Rightarrow P_0 = \left( \frac{1}{4}, 0 \right) \in C \Rightarrow f(P_0) = \frac{1}{16} – \frac{1}{8} = -\frac{1}{16}$

Esercizi su massimi e minimi in 2 variabili - per parametrizzare il bordo abbiamo bisogno di due curve
Il bordo non è una curva regolare pertanto bisogna usare due curve regolari per parametrizzarlo ${\gamma}_1$ e ${\gamma}_2}

parametrizzazione del bordo e teorema di Weierstress per individuare massimi e minimi

Il bordo dell’insieme può essere parametrizzato con due archi di circonferenza ${{\gamma }_{1}}$ e ${{\gamma }_{2}}$.

Tra i potenziali massimi e minimi bisogna prendere in considerazione anche l’intersezione tra ${{\gamma }_{1}}$ e ${{\gamma }_{2}}$ in ${{P}_{1}}=\left( 0,-1 \right)$ e ${{P}_{2}}=\left( 1,0 \right)$.

Il bordo del primo arco di circonferenza può essere parametrizzato con ${{\gamma }_{1}}=\left( \cos t,\sin t \right)$, dove $t\in \left[ 0,\frac{3}{2}\pi \right]$.

${{\phi }_{1}}\left( t \right)=f\left( \cos t,\sin t \right)={{\cos }^{2}}t+{{\sin }^{2}}t-\frac{\cos t}{2}=1-\frac{\cos t}{2}$

${{\phi }_{1}}^{\prime }\left( t \right)=\frac{\sin t}{2}=0$ $\Rightarrow $ $t=\left\{ 0,\pi \right\}$

Per $t=0$ si ha ${{P}_{2}}=\left( \cos 0,\sin 0 \right)=\left( 1,0 \right)$ , $f\left( {{P}_{2}} \right)=\phi \left( 0 \right)=\frac{1}{2}$

Per $t=\pi $ si ha ${{P}_{3}}=\left( \cos \pi ,\sin \pi \right)=\left( -1,0 \right)$, $f\left( {{P}_{3}} \right)=\phi \left( \pi \right)=\frac{3}{2}$

$f\left( {{P}_{4}} \right)=\phi \left( \frac{3}{2}\pi \right)=1$

Il bordo del secondo arco di circonferenza può essere parametrizzato traslando la curva ${{\gamma }_{0}}=\left( \cos t,\sin t \right)\,\,\,t\in \left( \frac{\pi }{2},\pi \right)$ di un vettore$c=\left( 1,-1 \right)$. Quindi si ottiene la parametrizzazione ${{\gamma }_{2}}={{\gamma }_{0}}+c=\left( \cos t+1,\sin t-1 \right)\,\,\,\,\,t\in \left( \frac{\pi }{2},\pi \right)$ .

${{\phi }_{1}}\left( t \right)=f\left( \cos t+1,\sin t-1 \right)={{\left( \cos t+1 \right)}^{2}}+{{\left( \sin t-1 \right)}^{2}}-\frac{\cos t+1}{2}=$

${{\cos }^{2}}t+2\cos t+1+{{\sin }^{2}}t-2\sin t+1-\frac{1}{2}\cos t-\frac{1}{2}=3+\frac{3}{2}\cos t-2\sin t$

Derivando rispetto a $t$ si ha:

${{\phi }_{1}}^{\prime }\left( t \right)=-\frac{3}{2}\sin t-2\cos t=0\,\,\,\Rightarrow \,\,\,\frac{\sin t}{\cos t}=-\frac{4}{3}\,\,\,\,\Rightarrow \,\,\tan t=-\frac{4}{3}$

La tangente è negativa nel secondo e quarto quadrante. Noi siamo interessati in particolare alla soluzione nel secondo quadrante, perché $t\in \left( \frac{\pi }{2},\pi \right)$ .

Il vettore $\left( \cos t,\sin t \right)$ , si ottiene normalizzando il vettore $v=\left( -3,4 \right)$, quindi $\left( \cos t,\sin t \right)=\frac{v}{\left\| v \right\|}=\left( -\frac{3}{5},\frac{4}{5} \right)$ .

Senza calcolare esplicitamente $t$ , è possibile ricavare le coordinate del punto sostituendo il valore di seno e coseno in ${{\gamma }_{2}}=\left( \cos t+1,\sin t-1 \right)$. Quindi ${{P}_{4}}=\left( -\frac{3}{5}+1,\,\,\,\frac{4}{5}-1 \right)=\left( \frac{2}{5},-\frac{1}{5} \right)$

Da cui $f\left( {{P}_{4}} \right)=\frac{4}{5}$ .

Il minimo e massimo assoluti sono:

$\underset{\left( x,y \right)\in \,\,C}{\mathop{\min }}\,\,f\left( x,y \right)=-\frac{1}{16}\,\,\,,\,\,\,\underset{\left( x,y \right)\in \,\,C}{\mathop{\max }}\,\,f\left( x,y \right)=\frac{3}{2}\,\,$

Esercizi su massimi e minimi in 2 variabili per insiemi costituiti da vincoli (Moltiplicatori di Lagrange/parametrizzazione del vincolo come una curva)

ESERCIZIO 5

Si vuole individuare punti di massimo e minimo della funzione$f\left( x,y \right)={{x}^{2}}y$ , lungo il vincolo definito in maniera implicita dalla parabola di equazione: $g\left( x,y \right)={{x}^{2}}+2x-y-2=0$

SOLUZIONE 1 (MOLTIPLICATORI DI LAGRANGE)

$\left\{ \begin{aligned} & 2xy = \lambda (2x + 2) \\ & x^2 = -\lambda \\ & x^2 + 2x – y – 2 = 0 \end{aligned} \right.$

Il vincolo è una parabola, non è un insieme limitato e quindi non vale il teorema di Weierstress, e allora i stazionari che individueremo sulla curva come zeri del gradiente della funzione Lagrangiana, andranno confrontati con i limiti per $x \to +\infty$ e $x \to -\infty$.

Faccio un passaggio di semplificazione nel sistema:

$\left\{ \begin{aligned} & xy = -x^2(x+1) \\ & \lambda = -x^2 \\ & x^2 + 2x – y – 2 = 0 \end{aligned} \right.$

Soluzione 1:

$\left\{ \begin{aligned} & x = 0 \\ & \lambda = 0 \\ & x^2 + 2x – y – 2 = 0 \end{aligned} \right.$

$\left\{ \begin{aligned} & x = 0 \\ & \lambda = 0 \\ & y = -2 \end{aligned} \right.$

Quindi $ P_0 = (0, -2) $.

Soluzione 2:

$\left\{ \begin{aligned} & y = -x(x+1) \\ & \lambda = -x^2 \\ & x^2 + 2x – y – 2 = 0 \end{aligned} \right.$

$\left\{ \begin{aligned} & y = -x(x+1) \\ & \lambda = -x^2 \\ & 2x^2 + 3x – 2 = 0 \end{aligned} \right.$

Il secondo sistema a sua volta ha due soluzioni:

$\left\{ \begin{aligned} & y = -\frac{3}{4} \\ & \lambda = -\frac{1}{4} \\ & x = \frac{1}{2} \end{aligned} \right. \quad \text{o} \quad \left\{ \begin{aligned} & y = -2 \\ & \lambda = -4 \\ & x = -2 \end{aligned} \right.$

Quindi $ P_1 = (-2, -2) $ e $ P_2 = \left(\frac{1}{2}, -\frac{3}{4}\right) $.

Infine, calcolando i valori della funzione $ f(x, y) $ nei punti $ P_0 $, $ P_1 $ e $ P_2 $ otteniamo:

$ f(0, -2) = 0, \quad f(-2, -2) = -8, \quad f\left(\frac{1}{2}, -\frac{3}{4}\right) = -\frac{3}{16} $

E l’andamento della funzione quando $ (x, y) $ tende all’infinito è $ +\infty $.

Si ha che ${{P}_{1}}$ è minimo assoluto, ${{P}_{0}}$è massimo relativo e ${{P}_{2}}$ è minimo relativo. Massimo assoluto non esiste perché la funzione va ad infinito percorrendo la parabola verso l’alto.

percorriamo una la curva cercando di capire lungo quali direzioni la funzione cresce
Le frecce colorate in verde indicano in quali direzioni la funzione cresce. $P_0$ è un punto di massimo relativo, perché è il punto di incontro tra due frecce che crescono in direzioni opposte

SOLUZIONE 2: PARAMETRIZZAZIONE DELLA PARABOLA

$f\left( x,y \right)={{x}^{2}}y$

$g\left( x,y \right)={{x}^{2}}+2x-y-2=0$

$y={{x}^{2}}+2x-2$

$\phi \left( x \right)=f\left( x,{{x}^{2}}+2x-2 \right)={{x}^{2}}\left( {{x}^{2}}+2x-2 \right)={{x}^{4}}+2{{x}^{3}}-2{{x}^{2}}$

${\phi }’\left( x \right)=4{{x}^{3}}+6{{x}^{2}}-4x=2x\left( 2{{x}^{2}}+3x-2 \right)=0,\,\,\,\Rightarrow \,\,{{x}_{0}}=-2\,\,,\,\,\,{{x}_{1}}=0,\,\,\,{{x}_{2}}=\frac{1}{2}$

$\underset{x\to \pm \infty }{\mathop{\lim }}\,\phi \left( x \right)=+\infty $,$\phi \left( -2 \right)=-8$, $\phi \left( 0 \right)=0$,$\phi \left( \frac{1}{2} \right)=-\frac{3}{16}$

ESERCIZIO 6

Individuare massimi e minimi assoluti di$f\left( x,y \right)={{e}^{{{x}^{2}}+2xy-{{y}^{2}}}}$ sul vincolo $C=\left\{ \left( x,y \right)\in {{\mathbb{R}}^{2}}:\frac{{{x}^{2}}}{4}+\frac{{{y}^{2}}}{6}=1 \right\}$

SOLUZIONE

Poiché l’esponenziale è una funzione monotona su tutto l’asse dei reali allora cercare massimi e minimi di $f\left( x,y \right)$ è lo stesso che cercare massimi e minimi di ${{f}_{1}}\left( x,y \right)={{x}^{2}}+2xy-{{y}^{2}}$

La Lagrangiana della funzione${{f}_{1}}\left( x,y \right)$ sull’insieme $C$ diventa $\Lambda \left( x,y,\lambda \right)={{x}^{2}}+2xy-{{y}^{2}}-\lambda \left( \frac{{{x}^{2}}}{4}+\frac{{{y}^{2}}}{6}-1 \right)$ .

Ponendo $\nabla \Lambda =0$ si ottiene il sistema:

$\left\{ \begin{aligned} & 2x+2y = \lambda \frac{x}{2} \\ & 2x-2y = \lambda \frac{y}{3} \\ & \frac{x^2}{4} + \frac{y^2}{6} = 1 \end{aligned} \right.$

Riorganizzando le prime due equazioni, otteniamo:

$\left\{ \begin{aligned} & (4-\lambda)x + 4y = 0 \\ & 6x – (\lambda + 6)y = 0 \\ & 3x^2 + 2y^2 = 12 \end{aligned} \right.$

Le prime due equazioni costituiscono un sistema lineare omogeneo $\mathbf{Ax}=\mathbf{0}$ , dove $\mathbf{A}$ è data da:

$\mathbf{A}=\left[ \begin{matrix}4-\lambda & 4 \\6 & -\lambda -6 \\\end{matrix} \right]$

Il sistema ammette soluzione se $\det \mathbf{A}=0$ , oppure $x=y=0$ . La seconda soluzione è da scartare perché non appartiene al vincolo $C$.

$\det \mathbf{A}={{\lambda }^{2}}+2\lambda -48=0\,\,\,\,\Rightarrow \,\,\,\,{{\lambda }_{1,2}}=\left\{ -8,6 \right\}$ . Da cui:

$\left\{ \begin{aligned} & \lambda = 6 \\ & -2x + 4y = 0 \\ & 3x^2 + 2y^2 = 12 \\ \end{aligned} \right.$

Da cui:

$\left\{ \begin{aligned} & \lambda = 6 \\ & x = 2y \\ & 14y^2 = 12 \\ \end{aligned} \right.$

${{P}_{0}}=\left( 2\sqrt{\frac{6}{7}},\sqrt{\frac{6}{7}} \right),{{P}_{1}}=\left( -2\sqrt{\frac{6}{7}},-\sqrt{\frac{6}{7}} \right)f\left( {{P}_{0}} \right)=f\left( {{P}_{1}} \right)={{e}^{6}}$

$\left\{ \begin{aligned} & \lambda = -8 \\ & 12x + 4y = 0 \\ & 3x^2 + 2y^2 = 12 \\ \end{aligned} \right.$

Da cui:

$\left\{ \begin{aligned} & \lambda = -8 \\ & y = -3x \\ & 21x^2 = 12 \\ \end{aligned} \right.$

Da cui:

$\begin{aligned} &P_2=\left( \frac{2}{\sqrt{7}},-\frac{6}{\sqrt{7}} \right), \\ &P_3=\left( -\frac{2}{\sqrt{7}},\frac{6}{\sqrt{7}} \right), \\ &f\left( P_2 \right)=f\left( P_3 \right)=e^{-8}\end{aligned}$

L’insieme $C$essendo rappresentato da un ellissi è chiuso e limitato e la funzione e continua su di esso e per Weierstress massimo e minimo esistono sicuramente e valgono $\underset{C}{\mathop{\max }}\,f\left( x,y \right)={{e}^{6}}$ e $\underset{C}{\mathop{\min }}\,f\left( x,y \right)={{e}^{-8}}$.

Le curve di livello della funzione $f\left( x,y \right)$ si ottengono ponendo costante l’esponente e sono delle iperboli.

grafico delle curve di livello della funzione sotto esame. Le frecce indicano in quale direzione le curve crescono

Il massimo e minimo della funzione su $C$ si hanno laddove l’ellissi risulta tangente all’iperbole. Quindi data la simmetria iperbolica della funzione si ha che ammette due massimi e due minimi.